Вы находитесь на странице: 1из 60

1.

A 10 year old boy was brought by her mother to come to hematology clinic for his routine
transfusion schedule since he was 1 year old. He alsoexperiencedeasily fatigue after doing
his routine activity since 1 month ago and experience limb edema. Since 4 years ago he
was suggested to take regular medication to decrease his iron overload, but his
compliance was poor. On physicalexamination he lookedmoderatelyill, nofever, short
stature, tachycardia, liverandspleenenlargement, and non pitting edema in low extremities.

What is the most appropriate examination to seek the cause of his new symptom?

a. Liver Ultrasonograph to find liver manifestation of iron overload


b. Echocardiography to find cardiac manifestation of iron overload
c. Kidney biopsy to find renal manifestation of iron overload
d. Genetic Analysis to find the genetic pattern inheritance
e. Screening for Hepatitis B and C to find the possibility of infection

2. A 7-year-old is diagnosed to have ß -thalassemia.

Which of the following is the most likely molecule affected in this patient?

a. Cell membrane
b. Cytoskeleton
c. Endoplasmic reticulum
d. Globin chain
e. Heme

3. Mr. Fadlan, 30 years old, was brought to the Emergency room with a chief complain
pruritic, erythematous skin rash over whole body and blisters on trunk and both arms since
4 days before admission.
He also got a not too high fever. One day before admission, he complained burning
sensation and redness on both eyes and erosion on his lips. There was history of taking
amoxicillin tablet and paracetamaol tablet before skin lesions appeared. Five months ago
he had suffered red itchy patches all over his body after he took ampicillin.
From physical examination was found 38.5 oC of body temperature, erosions on his lip,
genitalia, and erythema on his conjunctivae.
Dermatological status distribution : generalized, with erythematous macule, patches,
papules, vesicle, bulla, ptechiae, and purpura, on almost entire body.
Epidermolysis (+) : 7%, Diascopy test (+) for petechiae and purpura; Nikolsky sign (+),
Asboe Hansen sign (+).
Laboratory examinations : within normal limits
If the epidermolysis is positive, in what layer that the cleft formed
a. Epidermis
b. Dermis
c. Subcutan
d. Dermis and subcutan
e. Epidermis and subcutan
4. A 60-year-old woman is admitted to the ward with fever and night sweat since 2 months.
She had loss weight 10 kg for 2 months (from 50 kg to 40 kg). She has no cough. The
physician notes that there are lymph nodes palpated on her right neck and axilla 2-3 cm in
diameter, rubbery and not painful. There are no hepatosplenomegaly. Laboratory studies
shows: Hb 14 g/dL, Leucocyte 8500mm3, Platelet 260,000/mm3,

What is the most likely cause of her lymph node enlargement?

a. Malignancy of the breast


b. Infection of the head and neck area
c. Malignancy of the lymph node
d. Infection of the lymph node
e. Metastatic of cancer from nasopharyx

5. A 60-year-old male came with abdominal mass accompanied with fatigue, low grade
fever and night sweats since three months ago. Physical examination revealed low grade
fever and splenomegaly. There was no lymphadenopathy. Laboratory investigations
showed: hemoglobin 15 g/dL, WBC 36000 mm3, platelet 400.000 mm3. Peripheral blood
smear showed of all maturation series of granulocytes.

Which of the following is the most appropriate diagnosis?

a. Acute Myeloblastic Leukemia


b. Chronic Myelocytic Leukemia
c. Acute Lymphoblastic Leukemia
d. Multiple Myeloma
e. Anemia aplastic

6. A 5-year-old girl came to a private clinic with her mother complaining of sore throat since
3 days ago. Her mother told the doctor that her daughter had mild fever and cough during
the day. Physical examination showed temperature 38°C. HEENT examination showed
redness on her pharynx and large adenoid on both sides of the pharynx.

Which of the following best describespalatine tonsils?

a. It is a group of lymphoid tissue that is seen enlarged when infection happens.


b. It is located at the medial roof of nasopharynx.
c. It is located on each side of oropharynx between the palatoglossus arch and
palatopharyngeal arch.
d. It is located on each side of oropharynx between the palatoglossus arch and
palatopharyngeal arch, anterior to oropharyngeal isthmus.
e. It acts as primary lymphoid organs.

7. Mr. Irdan, 30-year-old male farmer came to the Emergency Department. He suffered from
fatigue, dizziness and physical weakness for the past 3 months. Recently he developed
shortness of breath on activity and palpitation. On physical examination he was pale. He
was diagnosed to have iron deficiency anaemia.

What is the abnormality that occur in this case?

a. High feritin
b. Low saturation transferin
c. low total binding capacity
d. High serum iron
e. High globin

8. A 27-year-old man develops high fever, chills, and headache for 5 days. After thorough
physical examination, the doctor checked the dengue serology test and complete blood
count. The lab results and physical examination show the patient has dengue fever, which
cause by dengue virus and the patients received supportive therapy in hospital.

Select the best statement that describe T-independent antibody production.

a. Antigens are mainly proteins from viruses, bacteria, foreign red blood cells, and
hapten-carrier molecules
b. Weaker immune response than for T-dependent antigens.
c. Need processed antigen by macrophages
d. Stronger immune response than for T-dependent antigens
e. The process involved IL-2

9. A 18 year old female complained of generalized weakness, breathlessness and


palpitations, and light headedness from the previous 6 months. She was having excessive
bleeding during menstruation. She was taking meals only once a day. The patient was
anemic, low of hemoglobin concentration, and low of serum iron.

What can you find in blood smear ?

a. Large or giant erythrocyte


b. Normosize of erythrocyte
c. Hypochrome of erythrocyte
d. Hyperchrome of erythrocyte
e. No correct answer
10. Which one of the followings is not allowed in Systemic Lupus Erythematosus criteria
based on American College Rheumatology:

a. Malar rash
b. Discoid lesion
c. Lupus hair
d. Positive ANA
e. Positive anti dsDNA

11. Which of the following complications is not likely to occur in SLE?

a. ALOPECIA
b. Butterly rash
c. Renal failure
d. Cerebral lupus
e. Leukaemia

12. All of the following statements concerning the lymphatic vessels are correct EXCEPT

a. the major route by which cancer metastasizes.


b. function to absorb large protein molecules and transport them to the bloodstream
c. serve as two-ways drainage toward the heart and return lymph to the bloodstream
through the thoracic duct or the right lymphatic duct.
d. carry lymphocytes from lymphatic tissues to the bloodstream.
e. are absent in the brain, spinal cord, eyeballs, bone marrow, splenic pulp, hyaline
cartilage, nails, and hair.

13. A 30 years old male patient was hospitalized with a severe anemia in District Hospital.
The patient had fatigue, pale and dizziness for one month and approximately 12 kg weight
lost for four months previous to admission. Severe iron deficiency anemia was diagnosed
in the patient by laboratory analyses. The stool examination revealed numerous eggs of
hookworm.

Which of following description best describe the worm's body part that causing anemia in
our patient?

a. An umbrella-like expansion of the cuticle


b. Copulatory bursa at the posterior end
c. Vulva at the ventral part of the body
d. Buccal capsule containing two pairs of teeth
e. The entire body of the worm

14. A 15-year-old man complained of enlargement in the left cervical lymph node since 2
months ago, which appear and disappear. Sometime he suffered pain.The biopsy was
performed for diagnosis. Microscopically could be seen lymph node follicle with a germinal
center reaction containing larger lymphocytes undergoing activation.

Which of the most likely cause of disease?

a. Foreign body reaction


b. Viral infection
c. Parasitic infection
d. Mycobacterial infection
e. Nonspecific infection

15. A 52-year-old man is admitted to ER with severe chest pain. Angiography demonstrates
a severe coronary occlusion. A thrombolytic agent is administered to reestablish perfusion.

The thrombolytic agent will most likely activate which of the following?

a. Heparin
b. Plasminogen
c. Thrombin
d. Kininogen
e. Prothrombin

16. The highest iron content in this food resources for nutritional support in Fe defficient is
.... ?

a. Meat
b. Organ meat
c. Spinach
d. Egg.
e. Milk

17. MicroRNA involves in normal and abnormal body processes. Which of the following is
the best statement that describes microRNAs (miRNAs)?

a. MicroRNAS stimulate influenza viral replication.


b. MiRNAs are absent in body fluid.
c. MiRNAs are small RNA molecules with less than 10 nucleotides length.
d. MiRNAs encode for protein.
e. MiRNAs regulate targetted genes through binding at the 3’ untranslated region.

18. A 12-year-old boy came to the clinic with the symptoms of fever and diarrhea. The
mother informed that he ate food bought from a food stall by the roadside. The doctor
diagnosed him with bacterial gastroenteritis.

Which of the following statements best describe evasion of bacteria from gastric juice?
a. Protection of bacteria by food particles
b. pH between 5-6 kill microbes and destroys most toxins
c. Low content of zinc and defensins to kill pathogens
d. H.pylori acidifies stomach acid and make gastritis and ulcer
e. Only E.coli can evade the gastric juice

19. What is the major cell component of the blood:

a. leucocytes
b. erythrocytes
c. plasma
d. thrombocyte
e. granulocyte

20. A 60-year-old male was found dead in his car parked at the porch of his house. It is
suspected that he committed suicide.

The affinity of hemoglobin for oxygen is increased by which of the following?

a. Metabolic acidosis
b. Hypercapnia
c. Hypoxemia
d. Anemia
e. Carbon monoxide poisoning

21. A 64 year old man went to hospital to medical check up examination. On physical
examination, he had hepatosplenomegaly. The laboratory results ; RBC 5 X 109/mm3, Hb 9
g/dL, WBC 245.400/mm3, Platelet 180 X 103/mm3.

The patient was diagnosed Chronic Myeloid Leukemia, and found Philadelphia
chromosome, which is caused of the translocation of chromosome :

a. 3 and 6
b. 6 and 9
c. 9 and 15
d. 9 and 22
e. 22 and 27

22. A 3-year-old boy diagnosed to have leukemia three months ago, was brought to the
emergency department for high fever since one week ago. His fever is accompanied by
increased fatigue, loss of appetite but no nausea or vomiting, and no weight loss.

Which of the following statement best describes primary lymphoid organ with the main
function of maturation of immune cells?

a. It is classified into red and white marrow.


b. Its superior part can expand to the oropharynx.
c. It is located lateral to pancreas
d. It is located at the posterior of manubrium sterni of thorax.
e. When it is enlarged it can block the upper airway passage.

23. In a newborn, which of the following iron binding glycoproteins that has an
antimicrobial effect?

a. Ferredoxin
b. Ferritin
c. Hemosiderin
d. Lactoferrin
e. Transferrin

24. A 10-year-old boy brought by his parent to primary health care service with the chief
complaint of redness in both eyes. It happens for about a week. He intensely rubbing his
eye because of itching sensation. There were also mucoid discharge from both eyes. From
ophthalmological examination revealed visual acuity of 6/6, conjunctival injection and
giant papil on tarsal conjunctiva for both eyes. What is the specific finding that help
conclude the diagnosis?

a. Conjunctival injection
b. Giant papil on the tarsal conjunctiva
c. Mucoid discharge
d. Redness of both eyes
e. Visual acuity of 6/6

25. A 3-year-old boy diagnosed to have leukemia three months ago, was brought to the
emergency department for high fever since one week ago. His fever is accompanied by
increased fatigue, loss of appetite but no nausea or vomiting, and no weight loss.

Which of the following statement best describes primary lymphoid organ with the main
function of maturation of immune cells?

a. It is classified into red and white marrow.


b. Its superior part can expand to the oropharynx.
c. It is located lateral to pancreas.
d. It is located at the posterior of manubrium sterni of thorax.
e. When it is enlarged it can block the upper airway passage.

26. A-27-year old woman come to the outpatient clinic complaining fatigue and easily feels
tired. Physical examination shows within normal limits except she has slightly increased
heart rate, pale conjunctiva but non icteric sclera. Laboratory examination revealed
hemoglobin 9 mg/dL, but other results have not available yet. She is given ferrous sulfate
325 mg twice daily and she is instructed to take the drug after meal.
What is the most appropriate reason of the instruction?

a. To increase the rate of drug absorption


b. To reduce abdominal discomfort
c. To avoid developing black stools
d. To facilitate drug distribution
e. To delay drug metabolism

27. A 27-year-old man develops high fever, chills, and headache for 5 days. After thorough
physical examination, the doctor checked the dengue serology test and complete blood
count. The lab results and physical examination show the patient has dengue fever, which
cause by dengue virus and the patients received supportive therapy in hospital.

Which of the following properties best describe T-cytotoxic cell?

a. Most are cd4+, recognize antigen on the cells’ surface via MHC class I, detached
from the cells when stimulating antigen is gone.
b. Most are cd8+, recognize antigen on the cells’ surface via MHC class I, detached
from the cell when stimulating antigen is gone.
c. Most are cd4+, recognize antigen on the cells’ surface via MHC class II, undergo
apoptosis when stimulating antigen is gone.
d. Most are cd8+, recognize antigen on the cells’ surface via MHC class I, undergo
apoptosis when stimulating antigen is gone.
e. Most are cd4+, recognize antigen on the cells’ surface via MHC class I, undergo
apoptosis when stimulating antigen is gone.

28. A 35-year-old woman presented with enlarged right axillary lymph node after viral
infection. The biopsy was performed. Macroscopically, the lymph node measured 1 x 0.8 x
0.5 cm, the cut surface white or gray, the capsule still intact. The microscopic examination
showed widening of the area between cortex and medulla of the lymph node

What are the most likely changes due to the above condition?

a. Reactive follicular hyperplasia


b. Paracortical hyperplasia
c. Sinus histiocytosis
d. Immunoblastic hyperplasia
e. Lymphoreticular hyperplasia

29. A 67-year-old male with chronic bronchitis is brought to the Emergency Department
exhibiting labored breathing and cyanosis.

The clinical sign of cyanosis is caused by which of the following?

a. An increase in the affinity of hemoglobin for oxygen


b. A decrease in the percent of red blood cells (hematocrit)
c. An increase in the concentration of carbon monoxide in the venous blood
d. A decrease in the concentration of iron in the red blood cells
e. An increase in the concentration of deoxygenated hemoglobin

30. A 64 year old man went to hospital to medical check up examination. On physical
examination, he had hepatosplenomegaly. The laboratory results ; RBC 5 X 109/mm3, Hb 9
g/dL, WBC 245.400/mm3, Platelet 180 X 103/mm3.

What kind of examination do you recommend?

a. Blood smear examination


b. Flow cytometry
c. Molecular
d. Cytogenetic
e. Cytokimia

31. Mr. Irdan, 30-year-old male farmer came to the Emergency Department. He suffered
from fatigue, dizziness and physical weakness for the past 3 months. Recently he
developed shortness of breath on activity and palpitation. On physical examination he was
pale. He was diagnosed to have iron deficiency anaemia.

The deficient agent of the above case is regulated by:

a. Blood PH
b. Hepcidin
c. Transferin receptor
d. Total Iron Binding Capasity
e. Mitochondria

32. A 12-year-old male had several lymphadenopathy on the left and right neck for 7
months. There were no changes in the size of lymph nodes after treated with penicillin. A
biopsy was conducted from one of the lymph node for histopathological diagnosis.
Microscopically, lymph node with an intact capsule, in several parts of tissue, a formation
of tubercle consisting of epitheloid cells could be found.

Which of the most likely cause of disease?

a. Mycobacterial infection
b. Viral infection
c. Parasitic infection
d. Fungal infection
e. Nonspecific infection
33. A histologic section shows an organ with a large amount of lymphatic tissue, including
nodules with germinal centers and a deeply invagi-nated mucosal surface formed by a
stratified but poorly defined epithelium.

The section is an example of:

a. Lymph node
b. Spleen
c. Palatine tonsil
d. Thymus gland
e. Peyer patch

34. Which one of the following is the correct statement .... ?

a. Patients diagnosed as AIDS if CD4 < 500


b. Post ARV must be given to all surgeons
c. Mother with HIV suggested to breastmilk her baby
d. Tenofovir is an ARV protease inhibitor
e. ARV is given after optimalized opportunistic infection therapy

34. Which one of the following is the correct statement .... ?

a. Patients diagnosed as AIDS if CD4 < 500


b. Post ARV must be given to all surgeons
c. Mother with HIV suggested to breastmilk her baby
d. Tenofovir is an ARV protease inhibitor
e. ARV is given after optimalized opportunistic infection therapy

35. Which of the following is a lymphatic organ located in the thoracic cavity just deep to
the sternum?

a. Peyer patch
b. . spleen
c. thymus
d. tonsil
e. vermiform appendix

36. A 25-year-old woman come to hospital with chief complaint of redness itchy patches
and some blister on her neck. For a week, she has redness itchy patches, and small blisters
on her neck, collar area which spread rapidly involving upper chest. Since two weeks ago
she has been using imitation neckless. Six months ago she suffered redness itchy patches
on her wrist after using new bracelet. Physical examination and laboratory findings are
within normal limit. The results of patch test were positive 3 (+++) to nickle.
Regarding to the Coombs and Gel classification of hypersensitivity, which of the following
is likely to occur in this patient?

a. Delayed type hypersensitivity (Type IV)


b. Contact hypersensitivity is characterized by granuloma
c. Type IV hipersensitivity is humoral type hypersensitivity
d. Contact hypersensitivity is characterized by tuberculoma
e. Type III hypersensitivity

37. A 35-year-old man came to the doctor with a mass in superior mediastinum.
Microscopic feature show cells medullary type and absent hasal corpuscle

Which is the best diagnosis?

a. Thymic hyperplasia
b. Benign thymoma
c. Malignant thymoma type I
d. Malignant thymoma type II
e. Squamous cell carcinoma

38. A 20-year-old male came to a primary health care service with the chief complaint of
redness with itchy sensation in both eyes. After thorough ophthalmological examination,
he was diagnosed of having allergic conjunctivitis.

What is the most possible type of hypersensitivity responsible for the condition?

a. Type II Hypersensitivity
b. Non hypersensitivity response
c. Type I Hypersensitivity
d. Type III Hypersensitivity
e. Type IV Hypersensitivity

39. The blood examination result of a 9-year-old girl with chief complain lost of appetite
and looks pale for 1 month was below:

Hb : 8.2 g/dL

Hct : 26 %

Leukocyte : 5.000 /mm3

Thrombocyte : 200.000 /mm3

Erythrocyte : 3.5 million/mm3

MCV : 60 fL
MCH : 24 pg

MCHC : 25 g/dL

What is your reason choosing the presumptive diagnosis as above?

a. The hemoglobin is low, MCH is low and erythrocyte is low


b. The hematocrit is low, MCV is low and MCHC is low
c. The hemoglobin is low, MCV is low and MCHC is low
d. The hematocrit is low, MCH is low and erythrocyte is low
e. The hemogloblin is low, the hematocrit is low and MCV is low

40. A mother with her newborn baby came to Puskesmas for post natal care. The doctor in
charge told the mother that her babies should get vaccination of Polio for protection. The
doctor said that the mother should follow the schedule for the other vaccinations in Kartu
Menuju Sehat in order to get the optimal results.

Which of the following is known as the number of antigen binding sites of an antibody?

a. Hapten
b. Ligand
c. Valence
d. Receptor
e. Epitopes

41. A 30 years old male patient was hospitalized with a severe anemia in District Hospital.
The patient had fatigue, pale and dizziness for one month and approximately 12 kg weight
lost for four months previous to admission. Severe iron deficiency anemia was diagnosed
in the patient by laboratory analyses. The stool examination revealed numerous eggs of
hookworm.

How did he acquire this infection?

a. Ingestion of water containing filariform larvae


b. Skin penetration by filariform larvae
c. Skin penetration by rhabditiform larvae
d. Ingestion of food contaminated with the eggs
e. Inhalation of dust carrying the cysts

42. A 10-year-old boy presented with a 3 days history of acute malaise, mild fever, and sore
throat. During physical examination, the doctor found tender, bilateral, cervical
lymphadenopathy. His pharynx, tonsils and buccal mucosa were red and inflamed and his
tonsils were studded with white areas of exudate. He was diagnosed as having acute
bacterial tonsillitis. Microbiological examination confirmed beta haemolytic streptococci
(Group A) infection.
Which of the following that least likely occurs in lymphoid tissue during above infection?

a. Differentiation of B-cell to plasma cells


b. Naive-B Cell activation and proliferation
c. Naive-NK Cell activation and proliferation
d. Naive-T Cell activation and proliferation
e. Presentation of antigen to T-helper cell via APC with MHC class II molecules

43. In acute hepatitis infection, which of the following antibody is raised the most?

a. IgA
b. IgD
c. IgE
d. IgG
e. IgM

44. A woman presents at the obstetrician's office for her second trimester evaluation.

Which of the following values would normally be less in the fetus than in the mother?

a. Hemoglobin concentration
b. Affinity of hemoglobin for oxygen
c. Erythrocyte binding of 2,3-BPG
d. Cardiac output/kg body weight
e. Cardiac glycogen content

45. A 27-year-old man develops high fever, chills, and headache for 5 days. After thorough
physical examination, the doctor checked the dengue serology test and complete blood
count. The lab results and physical examination show the patient has dengue fever, which
cause by dengue virus and the patients received supportive therapy in hospital.

Select the most appropriate statement which show the flow of antigen-antibody
interaction?

a. Activation of complement system, pathogen neutralization, opsonisation,


agglutination, PRR activation, antibody-dependent cell mediated immunity.
b. Activation of complement system, pathogen neutralization, agglutination,
inflammation, antibody-dependent cell mediated immunity.
c. Activation of complement system, pathogen neutralization, opsonisation,
agglutination, antibody-dependent cell mediated immunity.
d. Activation of complement system, pathogen neutralization, opsonisation,
agglutination, inflammation, antibody-dependent cell mediated immunity.
e. Activation of complement system, pathogen neutralization, opsonisation,
agglutination, inflammation, antibody-dependent cell mediated immunity, PRR
activation.
46. Which of the followings is the predominant cell type found in a germinal center?

a. Macrophage
b. B lymphocyte
c. Plasma cell
d. T lymphocyte
e. Reticular cell

47. 45 years old, male comes to the emergency room with chief complaint of fever. The
fever was started a week ago, but did not relief with antipyretic drugs. The fever happened
during evening till midnight. Other symptoms that accompany the fever was cough, with
muco-purulent sputum and mild-shortness of breath.

The doctor in charge instructs the nurse for obtain the patient's blood. Complete blood
count was ordered and the results as follow:

HGB : 15 g/dl
HCT : 45%
WBC : 25000/mm3
RBC : 5.6 million/mm3
PLT : 500000/mm3
MCV : 82 fL
MCH : 30 pg/cell
MCHC : 33 g/dL
ESR : 40 mm/h

What factor does the affect to decrease the ESR value?

a. RBC abnormalities: microcytic, spherocytosis, sickle cell


b. RBC abnormalities: rouleaux formation, macrocytic, sickle cell
c. Plasma abnormalities: fibrinogen, acute-phase protein, fasting
d. Plasma abnormalities: cholesterol, albumin, fibrinogen
e. RBC abnormalities: acute-phase protein, spherocytosis, sickle cell

47. 45 years old, male comes to the emergency room with chief complaint of fever. The
fever was started a week ago, but did not relief with antipyretic drugs. The fever happened
during evening till midnight. Other symptoms that accompany the fever was cough, with
muco-purulent sputum and mild-shortness of breath.

The doctor in charge instructs the nurse for obtain the patient's blood. Complete blood
count was ordered and the results as follow:

HGB : 15 g/dl
HCT : 45%
WBC : 25000/mm3
RBC : 5.6 million/mm3
PLT : 500000/mm3
MCV : 82 fL
MCH : 30 pg/cell
MCHC : 33 g/dL
ESR : 40 mm/h

What factor does the affect to decrease the ESR value?

a. RBC abnormalities: microcytic, spherocytosis, sickle cell


b. RBC abnormalities: rouleaux formation, macrocytic, sickle cell
c. Plasma abnormalities: fibrinogen, acute-phase protein, fasting
d. Plasma abnormalities: cholesterol, albumin, fibrinogen
e. RBC abnormalities: acute-phase protein, spherocytosis, sickle cell

48. Mr. Irdan, 30-year-old male farmer came to the Emergency Department. He suffered
from fatigue, dizziness and physical weakness for the past 3 months. Recently he
developed shortness of breath on activity and palpitation. On physical examination he was
pale. He was diagnosed to have iron deficiency anaemia.

What is the carrier of the deficient agent?

a. Hemoglobin
b. Transferrin
c. Ferritin
d. Bilirubin
e. Biliverdin

49. Which of the following statements is the most correct one?

a. Immunomodulator is the agent that helps in regulating immune system


b. Immunomodulator is the agent that helps in normalization immune system
c. Immunomodulator is the agent that helps in modulating immune system
d. Immunomodulator is the agent that helps in optimization immune system
e. All above statements are correct

50. Risk factor that develop IRIS is .... ?

a. Decreased conciousness
b. IDU (penasun)
c. Pregnancy
d. Very low CD4 before given ARV therapy
e. Age < 20 years
51. An 18-year-old woman came with gum bleeding since 1 week ago. The bleeding was
accompanied with weakness, fatigue and fever. Physical examination revealed pallor, fever,
skin ptechiae (+) of the upper and lower limbs. There was no hepatosplenomegaly and
lymphadenopathy. Laboratory investigations showed: hemoglobin 6.0 g/dL, WBC 1800
mm3, platelet 14.000 mm3. Blood smear showed normochromic-normocytic red cells with
very low of platelet and leukocyte count.

Which of the following is the most appropriate diagnosis?

a. Aplastic anemia
b. Acute Leukemia
c. Chronic Leukemia
d. Multiple Myeloma
e. Myeloproliferative neoplasm

52. What is the smallest cellular component of blood:

a. leucocytes
b. erythrocytes
c. plasma
d. thrombocyte
e. granulocyte

53. A 4-year-old boy came to clinic with itching in his inguinal every morning. The doctor
examined the patient and performed the cellophane procedure in his anal dimple and found
probable Enterobius vermicularis under microscope.

Which of the following process is most likely used to destroy large organisms such as
helminths?

a. Antibody dependent complement mediated killing


b. Antibody production
c. Antibody dependent cell mediated cytotoxixity
d. Opsonization
e. Neutralization

54. Which of the following that is the indication for using Immunostimmulant?

a. Immunodeficiency (HIV)
b. Infectious disease
c. Malignancy
d. all above are correct
e. none are correct
55. A 12-year-girl broughttotheemergency room transferred for rural hospital. She
wasreportedthat he developedpallor, dark urine, and jaundice over thepastfewdays. There
was no blood transfusion match for her. He has not exposed and hasnot beenexposedto a
jaundiced person. He had no history for taking antibiotic medication and blood
transfusion. The CBC in theofficeshows a low hemoglobinandhematocrit andelevationof
her bilirubin levels. Direct anti globulin test showed +4 major.

Which of the following is the most likely diagnosis of this patient's symptoms?

a. Thalassemia Intermedia
b. Autoimmune anemia Hemolytic
c. Iron deficiency anemia
d. Spherocytosis
e. Pyruvat Kinase Deficiency

56. Which of the following that is the work of Immunostimulant?

a. Enhances recognition of antigen by macrophage


b. Promotes production of IL-1 by APC
c. Promotes expression of IL-2 and other cytokines
d. Stimulates proliferation & differentiation of lymphocytes
e. C and D are correct

57. Some medical students are discussing about immunoglobulin molecules and
immunologic reactions.

a = Antigen bid to Fab of IgE.

b = Fc of IgE bind to mast cell membrane specific receptors.

c = Release of histamine.

d = Synthesis of IgE.

e = Vasodilatation and increase of blood vessel permeability.

Which of the following sequence is most likely describing the sequence of type I
hypersensitivity?

a. a-b-c-d-e
b. a-b-d-c-e.
c. b-a-c-d-e.
d. d-a-b-c-e.
e. d-b-a-c-e.

58. A 4-year-old previously well African American boy is brought to the office by his aunt.
She reports that he developed pallor, dark urine, and jaundice over the past few days. He
stays with her, has not travelled, and has not been exposed to a jaundiced person, but he is
taking trimethoprim sulfamet hoxazole for otitis media. The CBC in the office shows a low
hemoglobin and hematocrit, while his "stat" serum electrolytes, blood urea nitrogen (BUN),
and chemistries are remarkable only for an elevation of his bili rubin levels. His aunt seems
to recall his 8-year-old brother having had an "allergic reaction" to aspirin, which also
caused a short-lived period of anemia and jaundice.

Which of the following is the most likely cause of this patient's symptoms?

a. Hepatitis
b. Hepatitis A
c. Hemolytic-uremicsyndrome
d. Gilbert syndrome
e. Glucose-6-phosphate dehydrogenasedeficiency

59. A 10-year-old boy presented with a 3 days history of acute malaise, mild fever, and sore
throat. During physical examination, the doctor found tender, bilateral, cervical
lymphadenopathy. His pharynx, tonsils and buccal mucosa were red and inflamed and his
tonsils were studded with white areas of exudate. He was diagnosed as having acute
bacterial tonsillitis. Microbiological examination confirmed beta haemolytic streptococci
(Group A) infection.

The hyperemic pharynx and tonsils in the case are consequence of inflammatory response.
Which of the following shows consequences of inflammatory response at the first stage?

a. Antigen processing by macrophage


b. Margination of PMN
c. Neutrophil attraction by chemotactic factors
d. Phagocytosis of pathogen by PMN
e. Release of prostaglandine

60. An 8-year-old girl came to Paediatric Emergency Unit with difficulty in breathing with
wheezing as chief complaint and swelling on her face, fifteen minutes after analgetic
injection. The patient looked very weak.. Physical examination revealed: Her body weight
was 30 kg, height 140 cm. Blood pressure 69/45 mmHg, pulse 120x/minutes, regular, weak,
thready, respiratory rate 39x/minute. There are edematous face, edema on the eyelids,
signs of dyspnea (suprasternal, intercostal and epigastric retraction), wheezing and the
capillary refill >2 seconds.

At present, what is her diagnosis?

a. Anaphylaxis
b. Anaphylactic shock
c. Asma bronchiale + Angioedema
d. Asma bronchiale
e. Drug allergy

61. Mononuclear cells of white blood cells are divided into:

a. lymphocytes and monocytes


b. monocytes and neutrophils
c. monocytes and eosinophils
d. lymphocytes and basophils
e. neutrophils and eosinophils

62. A 10-year-old boy complains skin rash and diarrhoea directly after his first shrimp-
eating experience. Here are the first line of defence in priming for his problem:

a. MHC II and APC.


b. Th2 cells and APC.
c. CD4 Tcells and B cells.
d. Peristaltic movement and T cells.
e. Epithelial cells, mucus secretion and IgE.

63. A 64 year old man went to hospital to medical check up examination. On physical
examination, he had hepatosplenomegaly. The laboratory results ; RBC 5 X 109/mm3, Hb 9
g/dL, WBC 245.400/mm3, Platelet 180 X 103/mm3.

What will you find in the blood smear examination ?

a. Limphocyte
b. Eosinophill
c. Platelet
d. Myeloblast
e. Netrophyl

64. A 10-year-old patient was diagnosed with ß -thalassemia at the age of four years, after
presenting to his paediatrician with mild jaundice and anemia (haemoglobin level: 8.0 g/dl).
At the age of ten years, his physician recommended splenectomy.

What is the normal function of the lymphatic organ above?

a. Produce blood cells


b. Breakdown old RBC’s
c. Development of T-lymphocytes
d. Development of T cells
e. Production of lymphocytes

65. The percentage of hemoglobin saturated with oxygen will increase when which of the
following occurs?
a. The arterial PCO2 is increased
b. The hemoglobin concentration is increased
c. The temperature is increased
d. The arterial PO2 is increased
e. The arterial pH is decreased

66. A-72-year old man is diagnosed having Pernicious Anemia. The doctor needs to
prescribe him vitamin B12 but since the patient also takes proton pump inhibitor, the
doctor has to replace proton pump inhibitor with other drug.

What is the reason of this drug replacement?

a. The drug reduces vitamin B12 absorption


b. The drug ruins vitamin B12 distribution
c. The drug induces vitamin B12 metabolism
d. The drug inhibits vitamin B12 excretion
e. The drug inhibits vitamin B12 action

67. An 8 months infant was brought to emergency department with chief complain of pale.
Her mother had been noticed the symptom since 3 months ago. He also experienced
decrease of appetite and no longer active as he was. His 5 years old brother was
unremarkably healthy. There was no history of consumtion of specific drug. On physical
examination he looked severely ill, no fever, having failure to thrive, slightly jaundice,
murmur grade II/6 in allostia was found in cardiacauscultation. Liver and spleen
enlargement without lymphadenopathy were also noted.

Which of the following is the least likely differential diagnosis for this patient?

a. Autouimmunehemolytic anemia
b. Thalassemia beta major
c. Hemoglobinopathy
d. Deficiencyof 6 phosphatedehydrogenase
e. Irondeficiency anemia

68. Immune complex-mediated disorders - IgG IgM bind antigen - Antigen antibody
complexes deposit in tissues ? inflammation ? neutrophils and monocytes damage tissue

a. Type I Hypersensitivity
b. Type II Hypersensitivity
c. Type III Hypersensitivity
d. Type IV Hypersensitivity
e. Cell-mediated immunity
69. An 8-year-old boy came to primary health care service with complaint of lump at his
neck since 6 days ago accompanied with sore throat and fever. He was diagnosed with
pharyngitis with mild lymphadenopathy.

The thoracic duct of the lymphatic system will most likely empty into which of the
following?

a. Left subclavian vein


b. Lymphatic duct
c. Right subclavian vein
d. Internal jugular vein
e. Brachiocephalic vein

70. Which of the following is the most correct equipment for torniquette test .... ?

a. Kertas saring
b. Sphygmomanometer
c. Alcohol swab 70%
d. Sterile blood lancet
e. No correct answer

71. Which of the following that is not the indication for using Immunosuppressant?

a. Autoimmune disease
b. Infectious disease
c. Malignancy
d. Organ/tissue transplantation
e. Severe sepsis

72. A 25-year-old woman come to hospital with chief complaint of redness itchy patches
and some blister on her neck. For a week, she has redness itchy patches, and small blisters
on her neck, collar area which spread rapidly involving upper chest. Since two weeks ago
she has been using imitation neckless. Six months ago she suffered redness itchy patches
on her wrist after using new bracelet. Physical examination and laboratory findings are
within normal limit. The results of patch test were positive 3 (+++) to nickle.

Which of the following is the most likely diagnosis?

a. Contact urticaria
b. Irritant contact dermatitis
c. Atopic dermatitis
d. Allergic contact dermatitis
e. Seborrheic dermatitis
73. A 24-year-old women is admitted with lymphnode enlargement on the left neck since 1
week before. She had sore throat since 10 days before.

What is the most likely cause of her lymphnode enlargement?

a. Malignancy
b. Infection
c. Autoimmune
d. Iatrogenic
e. Allergic

74. The clinical laboratory staff performed APTT on 8-year-old boy who came with
prolonged bleeding after circumsition.

What is the purpose of APTT?

a. To evaluate only extrinsic pathway


b. To evaluate only intrinsic pathway
c. To evaluate only common pathway
d. To evaluate both extrinsic and common pathway
e. To evaluate both intrinsic and common pathway

75. A 12-year-old girl complains chronic rhinorrhea and nasal obstruction. All complains
relief after taking oral Cetirizine. Her grandparents have asthma. She feels better after
getting immunotherapy.

The CORRECT statement of the role of Treg cell for her better condition:

a. Developing TH2 cell homing to tissues.


b. Exerts regulatory function of Th2 in producing IL-4 and IL-13.
c. Exerts regulatory function on B cells by suppressing IgG4 and IgA.
d. Developing IL-10 and TGF-Beta in mast cell, basophil and eosinophil.
e. Exerts regulatory function on B cells by suppressing effector T cells and contribute
toIgG4 synthesis.

76. A mother with her newborn baby came to Puskesmas for post natal care. The doctor in
charge told the mother that her babies should get vaccination of Polio for protection. The
doctor said that the mother should follow the schedule for the other vaccinations in Kartu
Menuju Sehat in order to get the optimal results.

The adaptive immune responses different from the innate immune responses due to its
unlimited diversities. Such diversities happen due to........................

a. VJ recombination of T and B cells receptors


b. Variable germline
c. Pattern Recognition Receptors (PRRs)
d. HLA type I
e. HLA type II

77. Blood will coagulate if there is an injury of the blood vessel. Which of the following
process is most likely the first step of blood coagulation?

a. Constriction of injured blood vessel.


b. Exposed collagen binding with platelets.
c. Formation of platelet plugs.
d. Formation of red cloth.
e. Thromboxane release.

78. A 10-year-old boy brought by his parent to primary health care service with the chief
complaint of redness in both eyes. It happens for about a week. He intensely rubbing his
eye because of itching sensation. There were also mucoid discharge from both eyes. From
ophthalmological examination revealed visual acuity of 6/6, conjunctival injection and
giant papil on tarsal conjunctiva for both eyes.

What is the most typical corneal complication of the condition?

a. Keratitis
b. Corneal edema
c. Infected corneal ulcer
d. Peripheral corneal ulcer
e. Shield ulcer

79. A 30 years old male patient was hospitalized with a severe anemia in District Hospital.
The patient had fatigue, pale and dizziness for one month and approximately 12 kg weight
lost for four months previous to admission. Severe iron deficiency anemia was diagnosed
in the patient by laboratory analyses. The stool examination revealed numerous eggs of
hookworm.

By what mechanism does the worm causing his anemia condition?

a. Nutrition competition with the host


b. Intestinal wall’s blood sucked by cutting plate organ
c. Inflammation caused by intestinal immune response
d. lowering B12absorption in the intestine
e. Systemic inflammatory responses to the hookworm

80. A 5-year-old boy came to the clinic with abrupt onset of fever and runny nose. The
doctor diagnosed the boy with acute rhinitis due to viral infection. T cell receptor as
antigen receptor for spesific antigen are associated with major histocompatibility (MHC)
proteins on the surface of the cell. If a nucleated cell is infected by virus, it will express a
MHC class I molecules.
Which of the following cells most likely recognizesClass I MHC molecules?

a. Cytotoxic T cell
b. T helper 1
c. T helper 2
d. T regulator
e. Cd4+ T Cells

81. Prior to circumsition, doctor asked to perform laboratory screening test for a 4-year-old
boy.

The doctor performed Bleeding Time test. What is the method that performed on the
earlobe?

a. Ivy method
b. Duke method
c. Lee & White method
d. Tilt tube method
e. Direct method

82. A 24-year-old women is admitted with lymphnode enlargement on the left neck since 1
week before. She had sore throat since 10 days before.

Which of the following is the most likely diagnosis?

a. Lymphoma
b. Lymphadenitis tuberculosis
c. Reactive lymphadenopathy
d. Metastatic cancer to lymph node
e. Leukemia

83. A patient with left neck mass. The specimen consists of two lymph nodes.
Histopathology: The tumor cells are monotonous, are intermediate in size between small
lymphocytes and large non cleaved cells, with a high mitotic activity and a "starry sky"
pattern.

The most likely agent to be associated with this lymphoma is:

a. Tuberculosis
b. Mycosis
c. Epstein-Barr Virus
d. Ancylostomiasis
e. Toxoplasmosis

84. A 10-year-old boy presented with a 3 days history of acute malaise, mild fever, and sore
throat. During physical examination, the doctor found tender, bilateral, cervical
lymphadenopathy. His pharynx, tonsils and buccal mucosa were red and inflamed and his
tonsils were studded with white areas of exudate. He was diagnosed as having acute
bacterial tonsillitis. Microbiological examination confirmed beta haemolytic streptococci
(Group A) infection. The etiological agent of the case above has peptidoglycan layer on its
surface. Peptidoglycan has role as pathogen associated molecular patterns (PAMPs)
which could recognize by Pattern recognition Receptors (PRRs).

Which PRR below is the most likely to have the ablity to recognize the peptidoglycan?

a. TLR 1
b. TLR 2
c. TLR 4
d. TLR 7
e. TLR 9

85. Mr. Fadlan, 30 years old, was brought to the Emergency room with a chief complain
pruritic, erythematous skin rash over whole body and blisters on trunk and both arms since
4 days before admission.

He also got a not too high fever. One day before admission, he complained burning
sensation and redness on both eyes and erosion on his lips. There was history of taking
amoxicillin tablet and paracetamaol tablet before skin lesions appeared. Five months ago
he had suffered red itchy patches all over his body after he took ampicillin.

From physical examination was found 38.5 oC of body temperature, erosions on his lip,
genitalia, and erythema on his conjunctivae.

Dermatological status distribution : generalized, with erythematous macule, patches,


papules, vesicle, bulla, ptechiae, and purpura, on almost entire body.

Epidermolysis (+) : 7%, Diascopy test (+) for petechiae and purpura; Nikolsky sign (+),
Asboe Hansen sign (+).

Laboratory examinations : within normal limits

Which of Mr. Fadlan skin lesion was caused by blood extravasasion?

a. Erythematous macule
b. Erythematous patches
c. Erythematous papules
d. Bulae
e. Purpura

84. A 10-year-old boy presented with a 3 days history of acute malaise, mild fever, and sore
throat. During physical examination, the doctor found tender, bilateral, cervical
lymphadenopathy. His pharynx, tonsils and buccal mucosa were red and inflamed and his
tonsils were studded with white areas of exudate. He was diagnosed as having acute
bacterial tonsillitis. Microbiological examination confirmed beta haemolytic streptococci
(Group A) infection. The etiological agent of the case above has peptidoglycan layer on its
surface. Peptidoglycan has role as pathogen associated molecular patterns (PAMPs)
which could recognize by Pattern recognition Receptors (PRRs).

Which PRR below is the most likely to have the ablity to recognize the peptidoglycan?

a. TLR 1
b. TLR 2
c. TLR 4
d. TLR 7
e. TLR 9

85. Mr. Fadlan, 30 years old, was brought to the Emergency room with a chief complain
pruritic, erythematous skin rash over whole body and blisters on trunk and both arms since
4 days before admission.

He also got a not too high fever. One day before admission, he complained burning
sensation and redness on both eyes and erosion on his lips. There was history of taking
amoxicillin tablet and paracetamaol tablet before skin lesions appeared. Five months ago
he had suffered red itchy patches all over his body after he took ampicillin.

From physical examination was found 38.5 oC of body temperature, erosions on his lip,
genitalia, and erythema on his conjunctivae.

Dermatological status distribution : generalized, with erythematous macule, patches,


papules, vesicle, bulla, ptechiae, and purpura, on almost entire body.

Epidermolysis (+) : 7%, Diascopy test (+) for petechiae and purpura; Nikolsky sign (+),
Asboe Hansen sign (+).

Laboratory examinations : within normal limits

Which of Mr. Fadlan skin lesion was caused by blood extravasasion?

a. Erythematous macule
b. Erythematous patches
c. Erythematous papules
d. Bulae
e. Purpura

86. Which one of the followings is the precursor cell of a macrophage in the papillary layer
of the dermis?

a. Lymphocyte
b. Neutrophil
c. Macrocyte
d. Monocyte
e. Lipocyte

87. A 10-year-old boy brought by his parent to primary health care service with the chief
complaint of redness in both eyes. It happens for about a week. He intensely rubbing his
eye because of itching sensation. There were also mucoid discharge from both eyes. From
ophthalmological examination revealed visual acuity of 6/6, conjunctival injection and
giant papil on tarsal conjunctiva for both eyes.

What is the most likely diagnosis?

a. Atopic conjunctivitis
b. Atopic keratoconjunctivitis
c. Seasonal keratoconjunctivitis
d. Vernal conjunctivitis
e. Viral conjunctivitis

88. A 35-year-old man diagnosed as AIDS. The doctor recognised the treatment. Some ARV
had been chosen. The doctor gives him zidovudine 2 dd 300 mg.

Which of the following is the CORRECT statement regarding the drug?

a. Reverse Transcriptase inhibitors


b. Protease inhibitor
c. Integrase inhibitor
d. Entry inhibitor

89. Mr. Fadlan, 30 years old, was brought to the Emergency room with a chief complain
pruritic, erythematous skin rash over whole body and blisters on trunk and both arms since
4 days before admission.

He also got a not too high fever. One day before admission, he complained burning
sensation and redness on both eyes and erosion on his lips. There was history of taking
amoxicillin tablet and paracetamaol tablet before skin lesions appeared. Five months ago
he had suffered red itchy patches all over his body after he took ampicillin.

From physical examination was found 38.5 oC of body temperature, erosions on his lip,
genitalia, and erythema on his conjunctivae.

Dermatological status distribution : generalized, with erythematous macule, patches,


papules, vesicle, bulla, ptechiae, and purpura, on almost entire body.

Epidermolysis (+) : 7%, Diascopy test (+) for petechiae and purpura; Nikolsky sign (+),
Asboe Hansen sign (+).
Laboratory examinations : within normal limits

Which function of the skin disturbed in this disease

a. Protection
b. Synthesis of vitamin D
c. Sensation
d. Thermal regulator
e. Excretion

90. A 17-year-old medical student presented with patches of raised red skin covered by a
flaky white buildup on her knees and elbows. The patches enlarged and become itchy and
burning immediately before and during major exams.

Which is the most likely underlying cause of this disorder?

a. Hyperplasia of dermal cells


b. A longer keratinocyte cell cycle
c. Production of cytokine by infiltrating inflammatory cells
d. Microabscesses of the dermis
e. Abnormal microcirculation in the epidermis

91. A 7-year-old is diagnosed to have ß -thalassemia.

What is most likely peptide combination in this this patient?

a. Four ? chains
b. Three a and one d (delta) chains
c. Two a (alpha) and two ß (beta) chains
d. Two a and two d (delta) chains
e. Two a and two ? (gamma) chains

92. There are three types of PMN (granulocyte) named according to their staining
characteristics in blood films. They are:

a. neutrophils, eosinophils, lymphocytes


b. lymphocytes, eosinophils, basophils
c. monocytes, neutrophils, eosinophils
d. eosinophils, neutrophils, basophils
e. basophils, monocytes, neutrophils

93. An 8-year-old boy came to primary health care service with complaint of lump at his
neck since 6 days ago accompanied with sore throat and fever. He was diagnosed with
pharyngitis with mild lymphadenopathy.
The thoracic duct of the lymphatic system will most likely empty into which of the
following?

a. Left subclavian vein


b. Lymphatic duct
c. Right subclavian vein
d. Internal jugular vein
e. Brachiocephalic vein

94. A-9-monthsinfantwasbroughttoemergencydepartmentwithchiefcomplainof pale.


Hermother hadbeennoticedthesymptomsince 4 monthsago. He
alsoexperienceddecreaseofappetiteandnolongeractive as he was. His 5
yearsoldbrotherwasunremarkablyhealthy.Therewasnohistoryofconsumptionofspecificdrug.
On physicalexamination he lookedseverelyill, nofever, havingfailuretothrive,
slightlyjaundice, murmurgrade II/6 in all ostium was found in cardiac auscultation.
Liverandspleenenlargementwithoutlymphadenopathy were alsonoted.

Whichofthefollowingistheleastlikelydifferential diagnosis forthispatient?

a. Autoimmunehemolytic anemia
b. Thalassemia beta major
c. Spherocytosis
d. Deficiencyof 6 phosphatedehydrogenase
e. Irondeficiency anemia

95. A 29-year-old male presents with recurring episodes of edema accompanied by chills
and fever. A history reveals prolonged travel in the Far East and a diagnosis of parasites
that block the lymph vessels.

Which of the following characteristic of vessels best differentiates between lymphatic and
vascular systems?

a. Spontaneous vasomotor activity


b. Absorption of glucose from the interstitial fluid
c. Absorption of nutrients from the gastrointestinal (GI) tract
d. Backflow of fluid is prevented by valves
e. Endothelial cells form the vessel walls

96. Which of the following is the most correct equipment for bleeding test .... ?

a. Sphygmomanometer
b. Stethoscope
c. Marking pen
d. Kertas saring
e. No correct answer
97. A 27-year-old man develops high fever, chills, and headache for 5 days. After thorough
physical examination, the doctor checked the dengue serology test and complete blood
count. The lab results and physical examination show the patient has dengue fever, which
cause by dengue virus and the patients received supportive therapy in hospital.

Which of the following is the most likely receptor that could recognize the ssRNA viral
genome inside the cell?

a. TLR 4
b. TLR 7
c. TLR 9
d. TLR 2
e. TLR 1

98. A 27-year-old man develops high fever, chills, and headache for 5 days. After thorough
physical examination, the doctor checked the dengue serology test and complete blood
count. The lab results and physical examination show the patient has dengue fever, which
cause by dengue virus and the patients received supportive therapy in hospital.

Which of the following characteristics are the most likely produce in primary response to
viral antigen?

a. A monomer immunoglobulin, able to transfer via placenta, 80% of total circulating


antibodies, able to activate complement system
b. A monomer immunoglobulin, able to transfer via placenta, 10% of total circulating
antibodies, has no ability to activate complement system
c. A pentamer immunoglobulin, has no ability to transfer via placenta, 10% of total
circulating antibodies, able to activate complement system
d. A pentamer immunoglobulin, has no ability to transfer via placenta, 10% of total
circulating antibodies, has no ability to activate complement system
e. A dimer immunoglobulin, has no ability to transfer via placenta, 10% of total
circulating antibodies, able to activate complement system

99. A 15-year-old girl came to the clinic with red itchy skin soon after eating shrimp co-
accidentally. She noticed that she has an allergy to seafood.

Which of the following immune cell is most likely responsible for the quickest release of
histamine that causes symptoms above?

a. Mast cell
b. Lymphocyte
c. Eosinophil
d. Basophil
e. Neutrophil
100. Lamivudine is an ARV that classified as group .... ?

a. Entry inhibitor
b. NRTI
c. NNRTI
d. Integrase Inhibitor
e. Protease Inhibitor

101. Immunosuppressive agents can inhibit the humoral or cellular mediated immune
responses alone and some of them inhibit both of them.

Which of the following immunosuppressive that inhibits both humoral and cellular
mediated immune responses?

a. Cyclosporine
b. Corticosteroid
c. Tacrolimus
d. Sirolimus
e. Azathioprine

102. A 18 year old female complained of generalized weakness, breathlessness and


palpitations, and light headedness from the previous 6 months. She was having excessive
bleeding during menstruation. She was taking meals only once a day. The patient was
anemic, low of hemoglobin concentration, and low of serum iron.

What is the most likely diagnosis ?

a. Aplastic anemia
b. Chronic disease of anemia
c. Iron deficiency anemia
d. Megaloblastic anemia
e. No correct answer

103. The thymus is a primary lymphoid organ in that it supplies other lymphoid organs and
tissues. It is enclosed by a thin connective tissue capsule.

Which of the following statement is the most CORRECT for the function for immunological
organ above?

a. Development of immunocompetent B-cells.


b. Proliferation of clones of mature B-cells.
c. Developing immunological self-tolerance.
d. Secretion of hormones for B-cell development.
e. Regulating B cell release into the systemic.

104. A 7-year-old is diagnosed to have ß -thalassemia.


What is the most frequent genetic defect that cause this disease?

a. Equal crossing over of gene


b. Group base deletion
c. Missense mutation
d. Point mutation.
e. Unequal crossing over of gene

105. The production of new T lymphocyte in the thymus occurs in which of the following
thymic regions :

a. Deep medulla
b. Thymic nodules
c. Superficial cortex
d. Corticomedullary junction
e. Thymic corpuscles

106. A 30-year-old female went for jogging around a lake. Which of the following best
describes her oxyhemoglobin transport and dissociation?

a. Hemoglobin affinity for oxygen is increased


b. There is less loading of O2 at the alveolar-capillary level than normal.
c. Oxygen unloading is increased at the tissue level.
d. Hemoglobin saturation with oxygen is lower than normal at any PaO2.
e. Increased 2,3-bisphosphoglycerate (BPG) shifts dissociation curve to the right.

107. A-8-year old boy was brought to the Emergency Room due to high fever. He was
suddenly unconscious soon after antibiotic injection. Systolic blood pressure 60 in
palpations, heart rate 140 x/minutes and respiration rate 28x/minutes. He immediately was
given adrenaline injection. The paramedic was instructed to monitor his urination.

What is the most appropriate reason of giving this drug?

a. To decrease heart rate


b. To increase urine formation
c. To decrease respiration rate
d. To increase vital tissues perfusion
e. To stimulate and maintain consciousness

108. All of the following statements concerning the thoracic duct are correct EXCEPT

a. The thoracic duct begins in a majority of individuals as the abdominal confluence of


lymph trunks at the L1–2 vertebral level.
b. The confluence of lymph trunks receives lymph from six main lymphatic trunks.
c. In a small percentage of individuals, the abdominal confluence of lymph trunks is
represented as a dilated sac.
d. The thoracic duct traverses the aortic aperture of the diaphragm.
e. The thoracic duct terminates at the junction of the left internal jugular vein and left
subclavian vein (i.e., left brachiocephalic vein) at the base of the neck.

109. Prior to circumsition, doctor asked to perform laboratory screening test for a 4-year-
old boy.

The doctor performed Clot Retraction Test. What kind of blood sample needed for this test?

a. Capillary blood
b. Venous blood
c. Arterial blood
d. Anticoagulated blood
e. Blood with heparin

110. All of the following statements concerning the spleen are correct EXCEPT

a. largest of the lymphatic organs


b. associated posteriorly with the left 9th through 11th ribs
c. located retroperitoneally
d. normally, does not descend inferior to the costal region
e. varies considerably in size, weight, and shape

111. A-9-monthsinfantwasbroughttoemergencydepartmentwithchiefcomplainof pale.


Hermother hadbeennoticedthesymptomsince 4 monthsago. He
alsoexperienceddecreaseofappetiteandnolongeractive as he was. His 5
yearsoldbrotherwasunremarkablyhealthy.Therewasnohistoryofconsumptionofspecificdrug.
On physicalexamination he lookedseverelyill, nofever, havingfailuretothrive,
slightlyjaundice, murmurgrade II/6 in all ostium was found in cardiac auscultation.
Liverandspleenenlargementwithoutlymphadenopathy were alsonoted.

What is the most likely physical examination finding that showed that the patient had
severe anemia?

a. Tachycardia
b. Ejection systolic murmur grade II/6 in all the cardiac ostia
c. Liver enlargement
d. Spleen enlargement
e. Icteric sclera

112. A 27-year-old pregnant woman come to antenatal check up and was given iron tablets.
Which the following factors will most likely affect iron absorption in this patient?

a. Ferric iron is absorbed better than ferrous iron.


b. Formation of insoluble complexes with phosphate increases iron absorption.
c. Gastric acidity helps to keep iron in the ferrous state and soluble in the upper gut .
d. Non-haem iron is absorbed better than haem iron.
e. Vitamin D helps to keep iron in the ferrous state.

113. An 8-year-old boy came to primary health care service with complaint of lump at his
neck since 6 days ago accompanied with sore throat and fever. He was diagnosed with
pharyngitis with mild lymphadenopathy.

What is the most likely lymph node that is affected in this patient?

a. Suprasternal node.
b. Cervical node.
c. Axillary node.
d. Submandibular node.
e. Postauricular node.

114. A 60-year-old woman is admitted to the ward with fever and night sweat since 2
months. She had loss weight 10 kg for 2 months (from 50 kg to 40 kg). She has no cough.
The physician notes that there are lymph nodes palpated on her right neck and axilla 2-3
cm in diameter, rubbery and not painful. There are no hepatosplenomegaly. Laboratory
studies shows: Hb 14 g/dL, Leucocyte 8500mm3, Platelet 260,000/mm3,

What is the next most appropriate step in making the diagnosis?

a. Bone marrow aspiration


b. Bone marrow biopsy
c. Lymph node excision biopsy
d. Lymph node fine needle aspiration biopsy
e. Immunophenotyping

115. A 10-year-old girl came to the clinic with swelling and redness of her left foot due to
insect bite. The doctor diagnosed her with cellulitis. Certain cells are involved in detecting
the pathogenic bacteria.

The Pattern Recognition Receptor (PRR) which involves in detecting Pathogen Associated
Molecular Patterns (PAMP) can most likely be found on which of the following cells?

a. B cells
b. T cells
c. Dendritic cells
d. Defensins
e. NK Cells

116. An 8-year-old girl came to Pediatric clinic with fever as chief complaint. It was occured
since 2 months ago, accompanied with butterfly rash in the cheek and also some swollen
joints on the interdigities of her fingers. Laboratory examination revealed that haemoglobin
9 g/dL, leucocyte 2300/mm3, thrombocyte 86000/mm3, proteinuria +++ and antinuclear
antibody was positive.

The main pathogenesis of this case?

a. Hypersensitivity type I
b. Hypersensitivity type II
c. Hypersensitivity type III
d. Hypersensitivity type IV
e. Complement activation

117. Mr. Irdan, 30-year-old male farmer came to the Emergency Department. He suffered
from fatigue, dizziness and physical weakness for the past 3 months. Recently he
developed shortness of breath on activity and palpitation. On physical examination he was
pale. He was diagnosed to have iron deficiency anaemia.

Which organ has a primary role in stimulating the formation of red blood cell?

a. Kidney
b. Liver
c. Bone marrow
d. Spleen
e. Heart

118. A-9 monthsinfantwasbroughttoemergencydepartmentwithchiefcomplainof pale.


Hermother hadbeennoticedthesymptomsince 4 monthsago. He
alsoexperienceddecreaseofappetiteandnolongeractive as he was. His 5
yearsoldbrotherwasunremarkablyhealthy.Therewasnohistoryofconsumptionofspecificdrug.
On physicalexamination he lookedseverelyill, nofever, havingfailuretothrive,
slightlyjaundice, murmurgrade II/6 in all ostium was found in cardiac auscultation.
Liverandspleenenlargementwithoutlymphadenopathy were alsonoted.

What is the basic mechanism cause of pale in this patient?

a. Hypersensitivity type 2
b. Iron overload
c. Imbalance of globin chain that make precipitate on membrane erythrocyte
d. Deficiency of metabolism enzyme of erythrocyte
e. Chronic blood loss
119. On a laboratorium examination on CBC, a 1-year-old is noted to have low hemoglobin
level with a microcytic anemia, anisopoikolocytocis, and basophilic stippling A follow-up
hemoglobin electrophores is demonstrates an increased of concentration of hemoglobin
A2, Hb F, and high ferittin level.

Which of the following is the most likely diagnosis in this patient?

a. Irondeficiency
b. Beta-Thalassemiatrait
c. Beta-Thalassemiamajor
d. Chronicsystemicillness
e. Leadpoisoning

120. Some medical students are discussing about immunoglobulin molecules and
immunologic reactions. Which of the following condition is important for hapten, a small
organic molecule, to act as an antigen?

a. It must never be synthesized inside the human body.


b. Its molecular weight must be more than 5,000.
c. It must be combined with a carbohydrate.
d. It must be combined with a protein.
e. It must never been introduce to human body before.

121. A 10-year-old girl came to the clinic with swelling and redness of her left foot due to
insect bite. The doctor diagnosed her with cellulitis. Certain cells are involved in detecting
the pathogenic bacteria.

Which of the following event is least likely to be triggered by PRR?

a. Increased phagocytosis
b. Increased production of cytokines and inflammatory mediators such as interferons
c. Increased cell migration
d. Increased attacked cell replication
e. Changes in expression of molecules involved in T cell APC function

122. An 18-year-old man complained of enlargement of the left cervical lymph node since 2
months ago, Microscopic examination of lymph node biopsy was as follow:

What is the most likely diagnosis of the above case?

a. Diffuse large B cell lymphoma


b. Follicular lymphoma
c. Hodgkin Lymphoma
d. T cell lymphoma
e. Tuberculous lymphadenitis
123. A 10 year old boy was brought by her mother to come to hematology clinic for his
routine transfusion schedule since he was 1 year old. He alsoexperiencedeasily fatigue
after doing his routine activity since 1 month ago and experience limb edema. Since 4
years ago he was suggested to take regular medication to decrease his iron overload, but
his compliance was poor. On physicalexamination he lookedmoderatelyill, nofever, short
stature, tachycardia, liverandspleenenlargement, and non pitting edema in low extremities.

What is the most likely drug he should take regularly?

a. Iron chelating agent


b. Folic acid
c. Vitamin E
d. Sulfas Ferosus
e. Vitamin C

124. Mr. Fadlan, 30 years old, was brought to the Emergency room with a chief complain
pruritic, erythematous skin rash over whole body and blisters on trunk and both arms since
4 days before admission.

He also got a not too high fever. One day before admission, he complained burning
sensation and redness on both eyes and erosion on his lips. There was history of taking
amoxicillin tablet and paracetamaol tablet before skin lesions appeared. Five months ago
he had suffered red itchy patches all over his body after he took ampicillin.

From physical examination was found 38.5 oC of body temperature, erosions on his lip,
genitalia, and erythema on his conjunctivae.

Dermatological status distribution : generalized, with erythematous macule, patches,


papules, vesicle, bulla, ptechiae, and purpura, on almost entire body.

Epidermolysis (+) : 7%, Diascopy test (+) for petechiae and purpura; Nikolsky sign (+),
Asboe Hansen sign (+).

Laboratory examinations : within normal limits

In establishing mr. Fadlan diagnosis, how many sites ofmucous membrane at least must
be involved?

a. 1
b. 2
c. 3
d. 4
e. 5

125. A mother with her newborn baby came to Puskesmas for post natal care. The doctor
in charge told the mother that her babies should get vaccination of Polio for protection. The
doctor said that the mother should follow the schedule for the other vaccinations in Kartu
Menuju Sehat in order to get the optimal results.

What is the least likely to be the consequences of antigen-antibody binding complex?

a. Bacterial agglutination to enhance phagocytosis


b. Opsonization of antigen
c. Alternative pathway of complement activation to induce cell lysis
d. Virus neutralization
e. Inflammation

126. A 3-year-old boy diagnosed to have leukemia three months ago, was brought to the
emergency department for high fever since one week ago. His fever is accompanied by
increased fatigue, loss of appetite but no nausea or vomiting, and no weight loss.

Which of the following secondary lymphoid organs that are grouped as MALT ?

a. Peyer’s patches, thymus and skin.


b. Tonsils, skin, bronchus and pancreas
c. Thymus, tonsils and spleen.
d. Spleen, thymus and tonsils.
e. Appendix, tonsils and Peyer’s patches.

127. A 56-year-old man came to emergency hospital due to abrupt onset of fever for 2 days.
The patient told that he had been travelling to Sulawesi for a week ago and very exhausted
during travelling. The complete blood count revealed leukocytosis and high neutrophil
count.

Apoptosis, Protein Kinase R and Interferon are the immune responses that act to trigger
"cell-autonomous defenses". Which one of these pathogens that least likely induced that
response?

a. Toxoplasma gondii
b. Plasmodium spp
c. Most viruses
d. Candida albicans
e. Listeria monocytogenes

128. A mother with her newborn baby came to Puskesmas for post natal care. The doctor
in charge told the mother that her babies should get vaccination of Polio for protection. The
doctor said that the mother should follow the schedule for the other vaccinations in Kartu
Menuju Sehat in order to get the optimal results. Which class of human immunoglobulin
plays role in protection of mucous membrane by preventing attachment of organisms?

a. Ig G
b. Ig A
c. Ig M
d. Ig D
e. Ig E

129. A 15-year-old girl has had intermittent purpura, arthalgia, abdominal pain and
hematuria for the past 3 months.The cutaneous purpura appears typically on the extensor
surfaces of the extremities and the buttock. Diascopy test (+).

Which of the following describes pathogenesis of her skin lessions?

a. Hypersensitivity type I
b. Hypersensitivity type II
c. Hypersensitivity type III
d. Hypersensitivity type IV
e. Hypersensitivity type I-IV

130. An 18-year-old man complained of enlargement of the left cervical lymph node since 2
months ago, Microscopic examination of lymph node biopsy was as follow:

What is the name of the cell pointed by arrow?

a. B Cell lymphocyte
b. Immunoblast
c. Monocyte
d. Reed Sternberg cell
e. T Cell lymphocyte

131. A 7-year-old is diagnosed to have ß -thalassemia.

What is the commonest chromosome that is affected in this patient?

a. Chromosome 1
b. Chromosome 11
c. Chromosome 12
d. Chromosome 20
e. Sex chromosome

132. An 8-year-old girl came to Pediatric clinic with fever as chief complaint. It was occured
since 2 months ago, accompanied with butterfly rash in the cheek and also some swollen
joints on the interdigities of her fingers. Laboratory examination revealed that haemoglobin
9 g/dL, leucocyte 2300/mm3, thrombocyte 86000/mm3, proteinuria +++ and antinuclear
antibody was positive.

What is the diagnosis of this case?


a. Fever of unknown origin
b. Juvenile idiopathic arthritis
c. Systemic lupus erythematosus
d. Acute rheumatic fever
e. Dermatomyositis

133. Effect of TLRs activation to the cells after contact with the microorganism's protein:

a. Opsonize bacteria and suppress apoptosis.


b. Killing bacteria and suppress coagulation.
c. Activate complement cascades and suppress apoptosis.
d. Induce phagocytosis and activate complement cascades.
e. Implement proinflammatory signaling pathways and killing bacteria.

134. A 44-year-old female with a history of excessive menstrual bleeding for 7 days is
associated with increasing fatigue and cold extremities. Laboratory results showed
hemoglobin concentration: 6 g/dL.n

Which of the following conditions is most likely reduced in this condition?

a. Arterial PO2
b. Total arterial oxygen content
c. Dissolved oxygen content
d. Percent O2 saturation in the arterial blood
e. Oxygen extraction

135. All of the following statements concerning the lymph nodes are correct EXCEPT

a. are organized collections of lymphatic tissue permeated by lymph channels.


b. produce lymphocytes and plasma cells and fi lter the lymph.
c. trap bacteria drained from an infected area and contain reticuloendothelial cells and
phagocytic cells (macrophages) that ingest these bacteria.
d. are called lacteals in the villi of the small intestine, where they absorb emulsified
fat.
e. are hard and often palpable when there is a metastasis and are enlarged and tender
during infection.

136. Mr. Irdan, 30-year-old male farmer came to the Emergency Department. He suffered
from fatigue, dizziness and physical weakness for the past 3 months. Recently he
developed shortness of breath on activity and palpitation. On physical examination he was
pale. He was diagnosed to have iron deficiency anaemia.

Which of the following statements about the deficient agent distribution is CORRECT?

a. Cells cannot regulate their uptake of agent with changing agent’s content
b. Agent’s homeostasis is maintained in part by agent regulatory proteins binding to
agent-responsiveelements in mRNA
c. Agent overload cannot occur because very efficient excretory mechanisms are
available
d. Transferrin decreases in agent deficiency to facilitate storage of iron
e. In the early stages of agent’s depletion, serum ferritin levels rise rapidly as the agent
is released from storage forms.

137. The ARV therapy must be given for HIV patients with conditions .... ?

a. CD4 >500
b. CD4 <500
c. Undetected viral load
d. Allergic to ARV
e. Pregnancy

138. A 21-year-old woman, had intermitten low grade fever for the past 1 month . She also
had loss of hair, pain at her fingers and feet associated with fatique. Physical examination
showed redness at her cheek and ulcer at the palatum on her oral cavity with hypertension.
Laboratory investigations revealed: Hb 6.8 gr/dl, Trombosit 80.000/mm3, ANA test: Positif,
Homogen pattern. Urine prot ++++

What is the most appopriate diagnosis?

a. Reactive Artritis
b. Osteoartritis
c. Nephritic lupus
d. Tuberculosis
e. Thypoid fever

139. A 50-year-old man came to the clinic with an abscess on his back. The abscesses are
filled with neutrophils performing phagocytosis.

Which of the following is the most appropiate stages of phagocytosis?

a. Chemotaxis and adherence – ingestion – phagosome formation – phagolysosome


– formation of residual body – digestion – discharge
b. Chemotaxis and adherence – ingestion – phagosome formation – phagolysosome
–digestion – formation of residual body – discharge
c. Chemotaxis and adherence – phagosome formation – ingestion– phagolysosome
– formation of residual body – digestion – discharge
d. Chemotaxis and adherence – ingestion – digestion – phagosome formation –
phagolysosome – formation of residual body– discharge
e. Chemotaxis and adherence – digestion - ingestion – phagosome formation –
phagolysosome – formation of residual body– discharge
140. An 8 months infant was brought to emergency department with chief complain of
pale. Her mother had been noticed the symptom since 3 months ago. He also experienced
decrease of appetite and no longer active as he was. His 5 years old brother was
unremarkably healthy. There was no history of consumtion of specific drug. On physical
examination he looked severely ill, no fever, having failure to thrive, slightly jaundice,
murmur grade II/6 in allostia was found in cardiacauscultation. Liver and spleen
enlargement without lymphadenopathy were also noted.

Laboratorium examination was showed Hemoglobin: 4,6 g/dL, hematocrite: 12 %, and


slightly elevation of indirect bilirubin level. Peripheral blood smear was showed: erythrocyte
hypo cromicmicrocyter, leukocyte and trombocyte without any abnormality.

What further laboratory examination do you want to choose to confirm the diagnosis of the
patient?

a. Hemoglobin analysis (hemoglobin electrophoresis)


b. Fe serum level, feritin
c. Urine examination
d. Coomb test examination
e. Osmoticfragility

141. A 21-year-old woman, had intermitten low grade fever for the past 1 month . She also
had loss of hair, pain at her fingers and feet associated with fatique. Physical examination
showed redness at her cheek and ulcer at the palatum on her oral cavity with hypertension.
Laboratory investigations revealed: Hb 6.8 gr/dl, Trombosit 80.000/mm3, ANA test: Positif,
Homogen pattern. Urine prot ++++

What is the most appopriate treatment for this patient?

a. Corticosteroid
b. Parasetamol
c. Chloramphenicol
d. Colcisin
e. Non Steroid Anti inflamasi

142. A 35-year-old woman came to the hospital with malar rash and was diagnosed with
systemic lupus erythematosus

a. Type I Hypersensitivity
b. Type II Hypersensitivity
c. Type III Hypersensitivity
d. Type IV Hypersensitivity
e. Cell-mediated immunity
143. A biopsy specimen was taken from a 45-year-old female who presented with bulla on
her upper extremity. Pathology anatomy diagnosis was Pemphigus vulgaris.

Which of the following features is seen with the above condition?

a. Necrotic areas surrounded by a palisade of macrophages


b. Onion - skin lesions
c. Fibrinous purulent exudates
d. Intra epidermal blister with acantholytic cells
e. Subcorneal blister with acantholytic cells

. 45 years old, male comes to the emergency room with chief complaint of fever. The fever
was started a week ago, but did not relief with antipyretic drugs. The fever happened during
evening till midnight. Other symptoms that accompany the fever was cough, with muco-
purulent sputum and mild-shortness of breath.

The doctor in charge instructs the nurse for obtain the patient's blood. Complete blood
count was ordered and the results as follow:

HGB : 15 g/dl
HCT : 45%
WBC : 25000/mm3
RBC : 5.6 million/mm3
PLT : 500000/mm3
MCV : 82 fL
MCH : 30 pg/cell
MCHC : 33 g/dL
ESR : 40 mm/h

How does the ESR work ?

a. Based on 3 process: aggregation, sedimentation and packing


b. Based on 3 process: sedimentation, aggregation and packing
c. Based on 3 process: aggregation, packing and sedimentation
d. Based on 3 process: packing, aggregation and sedimentation
e. Based on 3 process: sedimentation, packing and aggregation

145. Which of the following white blood cells will most likely engulf pathogens?

a. Macrophages
b. T cells
c. B cells
d. Lymphocytes
e. Monocytes
146. Nutrient and non nutrient interaction influence the optimizing mineral absorption. This
component decrease Fe absorption .... ?

a. Protein food sources


b. Medication
c. Phytat
d. Vitamin C
e. Vitamin K

147. A 21-year-old woman, had intermitten low grade fever for the past 1 month . She also
had loss of hair, pain at her fingers and feet associated with fatique. Physical examination
showed redness at her cheek and ulcer at the palatum on her oral cavity with hypertension.
Laboratory investigations revealed: Hb 6.8 gr/dl, Trombosit 80.000/mm3, ANA test: Positif,
Homogen pattern. Urine prot ++++

What is the another information need to know?

a. Icteric disease
b. Urine output
c. Hemoptisis
d. Dyspnea
e. History of chest pain

148. A 7-year-old is diagnosed to have ß -thalassemia.

What is the most likely finding of Hb electrophoresis of this patient?

a. Decreased of HbA
b. Decreased of HbA2
c. Decreased of HbF
d. Increased of HbH
e. Increased of HbA

149. Ab mediated, IgG, IgM injure cells directly by promoting phagocytosis & lysis via
complement activation through Fc domain

a. Type I Hypersensitivity
b. Type II Hypersensitivity
c. Type III Hypersensitivity
d. Type IV Hypersensitivity
e. Cell-mediated immunity

150. A 42-year-old patient is scheduled for a surgery that will likely require blood
transfusion. Because the patient has a rare blood type, an autologous blood transfusion is
planned. Prior to surgery, 1500 mL of blood is collected.
The collection tubes contain calcium citrate, which prevents coagulation by which of the
following action?

a. Blocking thrombin
b. Binding factor XII
c. Binding vitamin K
d. Chelating calcium
e. Activating plasminogen

151. An immature neutrophil may first be recognized in a stained smear of red bone
marrow at which stage of granulopoiesis:

a. Myeloblast
b. Promyelocyte
c. Myelocyte
d. Metamyelocyte
e. Band cell

152. A 55 years old man undergoes major surgery, which is the most appropriate simple
and fast test should be done to confirm the major surgical risks .... ?

a. Torniquette test
b. Hess test
c. Bleeding time time
d. Rumple Leed test
e. No correct answer

153. A 10-year-old boy presented with a 3 days history of acute malaise, mild fever, and
sore throat. During physical examination, the doctor found tender, bilateral, cervical
lymphadenopathy. His pharynx, tonsils and buccal mucosa were red and inflamed and his
tonsils were studded with white areas of exudate. He was diagnosed as having acute
bacterial tonsillitis. Microbiological examination confirmed beta haemolytic streptococci
(Group A) infection.

Select the best statement to describe how lymphoid tissue proliferation occurs in this
case:

a. Activation of cd8+ T-cell by APC with MHC class II molecules


b. Activation of cd4+ T-cell by APC with MHC class II molecules
c. Activation of cd4+T-cell by APC with MHC class I molecules
d. Activation of cd8+ T-cell by APC with MHC class I molecules
e. Activation of T-helper cell by APC with MHC class I molecules

154. A 10 year old boy was brought by her mother to come to hematology clinic for his
routine transfusion schedule since he was 1 year old. He alsoexperiencedeasily fatigue
after doing his routine activity since 1 month ago and experience limb edema. Since 4
years ago he was suggested to take regular medication to decrease his iron overload, but
his compliance was poor. On physicalexamination he lookedmoderatelyill, nofever, short
stature, tachycardia, liverandspleenenlargement, and non pitting edema in low extremities.

What is the cause of iron overload in this patient?

a. Regular transfusion
b. Genetic mutation intron
c. Consume high contain iron supplement
d. Increase hepcidin level
e. Decrease eritropoesis

155. A 12-year-old boy came to the clinic with the symptoms of fever and diarrhea. The
mother informed that he ate food bought from a food stall by the roadside. The doctor
diagnosed him with bacterial gastroenteritis.

Which of the following statements best describe evasion of bacteria from gastric juice?

a. Complement system
b. Interferon
c. Neutrophils
d. Dendritic cells
e. Immunoglobulin

156. A 7-year-old is diagnosed to have ß -thalassemia.

What is the most likely pathogenesis of this disease?

a. Autoimmune disease
b. Cobalamin deficiency.
c. Folic acid deficiency.
d. Genetic mutation of globin chain.
e. Iron deficiency.

157. A 3-year-old boy diagnosed to have leukemia three months ago, was brought to the
emergency department for high fever since one week ago. His fever is accompanied by
increased fatigue, loss of appetite but no nausea or vomiting, and no weight loss.

Which of the following secondary lymphoid organs that are grouped as MALT?

a. Peyer’s patches, thymus and skin.


b. Tonsils, skin, bronchus and pancreas.
c. Thymus, tonsils and spleen.
d. Spleen, thymus and tonsils.
e. Appendix, tonsilsand Peyer’s patches.
158. A 9-year-old boy got itching 30 minutes after interacting with cats.

The characteristic that causes his problem:

a. Release of IL-5 and influx of eosinophils.


b. Mast cell degranulation and T cells activated.
c. T cells activated and release of histamine and protease.
d. Release of histamine and protease and influx of eosinophils.
e. Mast cell degranulation and release of histamine and protease.

159. Which of the following that is not the work of Immunosuppressant?

a. Blocks recognition of antigen by macrophage


b. Inhibits production of IL-1 by APC
c. Inhibits expression of IL-2 and other cytokines
d. Inhibits proliferation & differentiation of (T and B) lymphocytes
e. Inhibits production of antibodies

160. Mr. Fadlan, 30 years old, was brought to the Emergency room with a chief complain
pruritic, erythematous skin rash over whole body and blisters on trunk and both arms since
4 days before admission.

He also got a not too high fever. One day before admission, he complained burning
sensation and redness on both eyes and erosion on his lips. There was history of taking
amoxicillin tablet and paracetamaol tablet before skin lesions appeared. Five months ago
he had suffered red itchy patches all over his body after he took ampicillin.

From physical examination was found 38.5 oC of body temperature, erosions on his lip,
genitalia, and erythema on his conjunctivae.

Dermatological status distribution : generalized, with erythematous macule, patches,


papules, vesicle, bulla, ptechiae, and purpura, on almost entire body.

Epidermolysis (+) : 7%, Diascopy test (+) for petechiae and purpura; Nikolsky sign (+),
Asboe Hansen sign (+).

Laboratory examinations : within normal limits

The test done in the case by dislodgement of the epidermis by lateral pressure was called:

a. Epidermolysis
b. Diascopy test
c. Nicolsky sign
d. Asboe Hansen sign
e. Laboratory examination
161. One child with an often exposure to protozoa, viruses and bacteria will be in the
immunological activation. This will exert:

a. Inflammation and IL-10.


b. Th2 and allergic disease.
c. Th2 and CD4+ regulatory T cells.
d. CD4+ regulatory T cells and IL-10.
e. Inflammation and autoimmune disease.

162. The clinical laboratory staff performed APTT on 8-year-old boy who came with
prolonged bleeding after circumsition.

What are the reagents we use in performing PT?

a. Ortho Brain Thromboplastin


b. Calcium Chloride
c. Activated thrombofax
d. Fibrinogen
e. Thrombin

163. A 65-year-old male with mild cyanosis presented to his physician with pruritis and
nose bleeding. A blood test revealed a hematocrit of 62, leading to the diagnosis of
polycythemia vera. Treatment included aspirin to prevent thromboses and periodic
phlebotomy to reduce the hematocrit.

a. How does this procedure affect the patient?


b. Reduces blood viscosity
c. Increases arterial oxygen saturation
d. Reduces blood velocity
e. Increases cardiac output
f. Increases arterial oxygen content

164. Cell-mediated immune disorders , delayed hypersensitivity, (Th1, Th17, CD8+ CTLs),
Granuloma Formation (lymphocyte, macrophage (outer)

a. Type I Hypersensitivity
b. Type II Hypersensitivity
c. Type III Hypersensitivity
d. Type IV Hypersensitivity
e. Cell-mediated immunity

165. A patient complains for sneezing, rhinorrhea and nasal obstruction for three years
with the history of atopic disease. These are inflammatory cells and products that can be
found in that situation:
a. Leucotrienes, IL-4, IL-3 and cytokines.
b. Histamine, cytokines, IL-13 and tryptase.
c. Cytokines, IL-4, prostaglandin and histamine.
d. Tryptase, prostaglandin, leucotrienes and IL-3.
e. IL-13, basic proteins, leucotrienes and histamine.

166. Vegetarian especially vegan have higher risk for iron deficient anemia, although they
combine iron plant food resources.

The limitation of mineral plant food resources is.... ?

a. Low bioavailability of mineral food resources


b. Low quantity of mineral.
c. High influence by food processing.
d. Low digestibility.
e. Competitive absorption with another mineral

167. A 25-year-old male came with gum bleeding since 1 week ago. The bleeding was
accompanied with weakness, fatigue, fever and unspecified bone pain. Physical
examination revealed pallor, fever there was multiple skin ptechiae, non tender lymph node
enlargement at regio colli/neck region and splenomegaly. Laboratory investigations
showed: hemoglobin 6.0 g/dL, WBC 23000 mm3, platelet 10.000 mm3.

Which of the following is the most likely morphology finding seen in his peripheral blood
smear?

a. All maturation series of granulocytes.


b. Shift to the left with mostly limfoblast dominant
c. Normochrom, normocyter with very low of platelet and leukocyte count
d. Normochrom, normocyter, normal wbc, plasma cell (+).
e. Shift to the left until myelocyte

168. A 25-year-old woman come to your clinic with chief complain of redness patches after
using topical lotion on her face. She also had rhinitis allergic from her past history.

Which of the most appropriate cell is contributing the condition above?

a. Mast cell
b. Melanocyte
c. Lymphocyte
d. Merkel's cell
e. Langerhans cell

169. What is the classification of HIV patients with lung tuberculosis and CD4 256 .... ?

a. I
b. II
c. III
d. IV
e. AIDS

170. A-27-year old woman come to the outpatient clinic complaining fatigue and easily
feels tired. Physical examination shows within normal limits except she has slightly
increased heart rate, pale conjunctiva but non icteric sclera. Laboratory examination
revealed hemoglobin 9 mg/dL, but other results have not available yet. She is given ferrous
sulfate 325 mg twice daily and she is instructed to take the drug after meal.

What is the most possible adverse effect might be occurred?

a. Excessive thirsty
b. Bloody stool
c. Constipation
d. Dizziness
e. Fatigue

171. Which of the following components in immune system occurs first upon invasion by a
virus?

a. Activation of T killer lymphocytes.


b. Activation of B lymphocytes.
c. Increase in tumour necrosis factor alpha.
d. Mobilisation of complement protein.
e. Opsonisation

172. The clinical laboratory staff performed APTT on 8-year-old boy who came with
prolonged bleeding after circumsition.

Doctor suggested substitution test to a 5-year-old boy, because of prolonged bleeding after
tooth extraction. He has a "bleeder's history among the male family members".

Susbtitution test result is as follows:

APTT: 70 sec (CN: 24-45); PT: 12 sec (CN: 10-14);

The most likely diagnosis of this boy is:

a. Factor V deficiency
b. Factor VII deficiency
c. Factor VIII deficiency
d. Factor X deficiency
e. Factor XII deficiency
173. A 30 years old male patient was hospitalized with a severe anemia in District Hospital.
The patient had fatigue, pale and dizziness for one month and approximately 12 kg weight
lost for four months previous to admission. Severe iron deficiency anemia was diagnosed
in the patient by laboratory analyses. The stool examination revealed numerous eggs of
hookworm.

What is appropriate medication taken to treat the patient's condition?

a. Antihelminthic agent, blood transfusion, and fluid replacement


b. Antihelminthic agent, blood transfusion, and iron supplement
c. Antihelminthic agent, blood transfusion, and B12 supplement
d. Antihelminthic agent, blood transfusion, and folic acid supplement
e. Antihelminthic agent, blood transfusion, and protein supplement

174. An 8-year-old boy came to primary health care service with complaint of lump at his
neck since 6 days ago accompanied with sore throat and fever. He was diagnosed with
pharyngitis with mild lymphadenopathy.

What is the most likely lymph node that is affected in this patient?

a. Suprasternal node
b. Cervical node
c. Axillary node
d. Submandibular node
e. Postauricular node

175. A 40-year-old man came to the hospital with anemia and received blood transfusion.
He suddenly developed shortness of breath and jaundice. The doctor then noticed that he
was transfused the wrong type of blood.

a. Type I Hypersensitivity
b. Type II Hypersensitivity
c. Type III Hypersensitivity
d. Type IV Hypersensitivity
e. Cell-mediated immunity

176. A developing erythrocyte and a developing neutrophil in red bone marrow may be
differentiated from one another by the presence of:

a. Golgi apparatus
b. Granules
c. Mitochondria
d. Peroxisomes
e. Rough endoplasmic reticulum (rER)
177. Inflammatory reactions involve the release of many cytokines. Which of the following
cytokines is most likely increased in mycobacterium infection?

a. Interleukin-1
b. Interleukin-22
c. Interleukin-18
d. Interleukin-6
e. Tumour necrosis factor alpha

178. Some medical students are discussing about immunoglobulin molecules and
immunologic reactions. Which of the following is the most likely appropriate for describing
the term "variable" in the variable regions of immunoglobulin molecules?

a. Ability to bind antigens


b. Amino acids sequence.
c. Carbohydrate content.
d. Protein chains molecular weight.
e. Three dimensional structures.

179. On a laboratorium examination on CBC, a 1-year-old is noted to have low hemoglobin


level with a microcytic anemia, anisopoikolocytocis, and basophilic stippling A follow-up
hemoglobin electrophores is demonstrates an increased of concentration of hemoglobin
A2, Hb F, and high ferittin level.

What is the most likely comprehensive management according the problems of this patient
above?

a. Transfusion of PRC 10-15 cc/kg, iron chelationa gent, education to increase the
compliance of the drug
b. Transfusion of PRC 10 cc/kg, iron supplementation, folatacid supplementation,
education to increase the compliance of the drug
c. Transfusion of PRC 10 cc/kg, folat supplementation
d. Transfusion of PRC 10-15 cc, iron chelation agent, folatacid supplementation,
education to increase the compliance of the drug
e. Transfusion of PRC 10-15 cc/kg

180. A 24-year-old women is admitted with lymphnode enlargement on the left neck since 1
week before. She had sore throat since 10 days before.

What is the most likely characteristic found on her physical examination of the
lymphnode?

a. Location at jugular chain, pain, soft


b. Location at jugular chain, pain, rubbery
c. Location at jugular chain, pain, hard
d. Location at posterior cervical, pain, soft
e. Location at posterior cervical, pain, rubbery

181. Laboratory examinationwasshowedhemoglobin : 4,6 g/dL, hematocrite: 12 %,


andslightlyelevationofindirect bilirubin level.
Peripheralbloodsmearwasshowed:erythrocytehypocromic microcyter,
leukocyteandtrombocytewithoutanyabnormality.

Whatfurtherlaboratoryexaminationdoyouwanttochoosetoconfirmthe diagnosis
ofthepatient?

a. Hemoglobin analysis (hemoglobin electrophoresis)


b. Fe serum level, feritin
c. Urine examination
d. Coomb test examination
e. Osmoticfragility

182. Many substances can acts as antigens. Which of the following substance is most
likely has the highest antigenicity property?

a. Lipid
b. Metal
c. Monosaccharide
d. Nucleic acid base
e. Protein

183. Lymphatic nodules with germinal centers are routinely noted in the:

a. Deep cortex of a lymph node


b. Red pulp of the spleen
c. Medulla of the thymus gland
d. Palatine tonsil of the pharynx
e. Medulla of a lymph node

184. An 18-year-old woman came with gum bleeding since 1 week ago. The bleeding was
accompanied with weakness, fatigue and fever. Physical examination revealed pallor, fever,
skin ptechiae (+) of the upper and lower limbs. There was no hepatosplenomegaly and
lymphadenopathy. Laboratory investigations showed: hemoglobin 6.0 g/dL, WBC 1800
mm3, platelet 14.000 mm3. Blood smear showed normochromic-normocytic red cells with
very low of platelet and leukocyte count.

Which of the followings is the most likely morphology finding in her bone marrow?

a. Hypercellular marrow
b. Normal bone marrow
c. Hypoplastic marrow with fat cell infiltration
d. Amegakariocyte marrow
e. Hyperplasia erythrocyte series

185. A-40-year old woman come to allergy clinic complaining of sneezing and nasal
congestion. Her mucosal nasal is appeared congested. She has been suffering from these
conditions since she was 10 years old. The doctor prescribes a corticosteroid nasal spray,
oral antihistamine, and decongestan

Which of following is mechanism of the nasal spray?

a. Causing vasoconstriction of blood vessels in nasal passages


b. Reducing the production of inflammatory mediators
c. Reducing the vagal afferent signal
d. Reducing the histamine release
e. Reducing mucous production

186. 45 years old, male comes to the emergency room with chief complaint of fever. The
fever was started a week ago, but did not relief with antipyretic drugs. The fever happened
during evening till midnight. Other symptoms that accompany the fever was cough, with
muco-purulent sputum and mild-shortness of breath.

The doctor in charge instructs the nurse for obtain the patient's blood. Complete blood
count was ordered and the results as follow:

HGB : 15 g/dl
HCT : 45%
WBC : 25000/mm3
RBC : 5.6 million/mm3
PLT : 500000/mm3
MCV : 82 fL
MCH : 30 pg/cell
MCHC : 33 g/dL
ESR : 40 mm/h

Why does the ESR increase in inflammation process?

a. The acute-phase protein will diminish the zeta-potential on the RBC surface, and
make the aggregation process easily (rouleaux formation) and shrink
b. The acute-phase protein will increase the content of plasma, which made the RBC
easily to shrink
c. The acute-phase protein will replace the content of plasma, therefore, the RBC will
easily to shrink.
d. The acute-phase protein will diminish the zeta-potential on the RBC surface and
activate the coagulation process, then made the RBC easily to clump and shrink.
e. The acute-phase protein will increase the content of plasma and diminish the zeta-
potential on RBC surface, and make agglutination of RBC through coagulation
activation

186. 45 years old, male comes to the emergency room with chief complaint of fever. The
fever was started a week ago, but did not relief with antipyretic drugs. The fever happened
during evening till midnight. Other symptoms that accompany the fever was cough, with
muco-purulent sputum and mild-shortness of breath.

The doctor in charge instructs the nurse for obtain the patient's blood. Complete blood
count was ordered and the results as follow:

HGB : 15 g/dl
HCT : 45%
WBC : 25000/mm3
RBC : 5.6 million/mm3
PLT : 500000/mm3
MCV : 82 fL
MCH : 30 pg/cell
MCHC : 33 g/dL
ESR : 40 mm/h

Why does the ESR increase in inflammation process?

a. The acute-phase protein will diminish the zeta-potential on the RBC surface, and
make the aggregation process easily (rouleaux formation) and shrink
b. The acute-phase protein will increase the content of plasma, which made the RBC
easily to shrink
c. The acute-phase protein will replace the content of plasma, therefore, the RBC will
easily to shrink.
d. The acute-phase protein will diminish the zeta-potential on the RBC surface and
activate the coagulation process, then made the RBC easily to clump and shrink.
e. The acute-phase protein will increase the content of plasma and diminish the zeta-
potential on RBC surface, and make agglutination of RBC through coagulation
activation

187. A 65-year-old slightly cyanotic male presents to his physician complaining of pruritis
and nose bleeds. A blood test reveals hematocrit of 62%, leading to the diagnosis of
polycythemia vera. Treatment includes aspirin to prevent thromboses and periodic
phlebotomy to reduce the hematocrit.

The reduction in hematocrit is beneficial because it does which of the following?

a. Reduces blood viscosity


b. Increases arterial oxygen saturation
c. Reduces blood velocity
d. Increases cardiac output
e. Increases arterial oxygen content

188. A 56-year-old man came to emergency hospital due to abrupt onset of fever for 2 days.
The patient told that he had been travelling to Sulawesi for a week ago and very exhausted
during travelling. The complete blood count revealed leukocytosis and high neutrophil
count.

Complement activation lead to recruitment of inflammatory cell, opsonization of pathogens


and killing of pathogens. Which pathway in complement activation will be triggered by
combination of adaptive and innate immune responses?

a. Classical pathway
b. MB-Lectin pathway
c. Alternative pathway
d. Combination of classical and MB-Lectin pathway
e. Combination of MB-Lectin and Alternative pathway

189. A 30 years old male patient was hospitalized with a severe anemia in District Hospital.
The patient had fatigue, pale and dizziness for one month and approximately 12 kg weight
lost for four months previous to admission. Severe iron deficiency anemia was diagnosed
in the patient by laboratory analyses. The stool examination revealed numerous eggs of
hookworm.

What is the best prevention method told to the community for stopping the disease
transmission?

a. Proper toilet hygiene and washing hands before meal


b. Proper toilet hygiene and cutting nail regularly
c. Proper toilet hygiene and covering meals when not in use
d. Proper toilet hygiene and proper trash care
e. Proper toilet hygiene and using proper footwear outside house

190. The blood examination result of a 9-year-old girl with chief complain lost of appetite
and looks pale for 1 month was below:

Hb : 8.2 g/dL
Hct : 26 %
Leukocyte : 5.000 /mm3
Thrombocyte : 200.000 /mm3
Erythrocyte : 3.5 million/mm3
MCV : 60 fL
MCH : 24 pg
MCHC : 25 g/dL
What is the presumptive diagnosis for this patient based on the result?

a. Anemia normochromic normocytic


b. Anemia hypochromic normocytic
c. Anemia hypochromic microcytic
d. Anemia normochromic macrocytic
e. Anemia normochromic microcytic

191. A 60-year-old female noticed the sudden appearance of multiple petechiae on her
extremities and mild epistaxis. She had no other symptoms and CBC performed by her
primary-care physician identified an isolated thrombocytopenia with a platelet count of 30
x 109/L. She was given a diagnosis of immune thrombocytopenic purpura (ITP)

What can you find in blood smear ?

a. Large or giant thrombocyte


b. Normosize of thrombocyte
c. Hypochrome of thrombocyte
d. Normochrome of thrombocyte
e. No correct answer

192. A 5-year-old girl came to a private clinic with her mother complaining of sore throat
since 3 days ago. Her mother told the doctor that her daughter had mild fever and cough
during the day. Physical examination showed temperature 38.°C. HEENT examination
showed redness on her pharynx and large adenoid on both sides of the pharynx.

Which of the following best describes palatine tonsils ?

a. It is a group of lymphoid tissue that is seen enlarged when infection happens.


b. It is located at the medial roof of nasopharynx.
c. It is located on each side of oropharynx between the palatoglossus arch and
palatopharyngeal arch.
d. It is located on each side of oropharynx between the palatoglossus arch and
palatopharyngeal arch, anterior to oropharyngeal isthmus.
e. It acts as primary lymphoid organs.

193. Bleeding time is an test for .... ?

a. Hemostasis
b. Homeostasis
c. Homeoreology
d. Hemoconcentration
e. No correct answer

194. The cisterna chyli is the inferior end of which of the following structures?
a. inferior vena cava
b. abdominal aorta
c. renal vein
d. testicular artery
e. thoracic duct

195. A 60-year-old female noticed the sudden appearance of multiple petechiae on her
extremities and mild epistaxis. She had no other symptoms and CBC performed by her
primary-care physician identified an isolated thrombocytopenia with a platelet count of 30
x 109/L. She was given a diagnosis of immune thrombocytopenic purpura (ITP)

What can you find in bone marrow?

a. A few of megakaryocyte
b. Numerous megakaryocyte
c. Increase of erythrocyte activity
d. Decrease of erythrocyte activity
e. No correct answer

196. A 65-year-old smoker develops squamous cells bronchogenic carcinoma, which


metastasizes to the tracheobronchial and parasternal lymph nodes.

Which of the following best describes cancer cells metastasize in this patient?

a. Increased capillary pressure


b. Increased capillary permeability
c. High interstitial protein concentration
d. Increased capillary oncotic pressure
e. Increased central venous pressure

197. A 30-year-old man come to hospital with chief complaint of red itchy patches and
blister on his trunk, left and right forearm which spreaded rapidly, involving almost every
part of his body. He also complained erosion on his lips, and anus, red and watery eyes,
fatigue and fever. Ten days earlier, he had sore-throat and took cotrimoxazole and
paracetamol. Six months ago he had also suffered red itchy circumscribed patches with
central banching on his body after taking paracetamol. Dermatologic examination: There
were generalised erythematous macules, papules, and plaque on his face, trunk and
extremities. There were blisters, pustules, erosions, excoriations, crusts, and scales on the
trunk and both legs. Nikolsky sign test was negative.

Which of the following is the most likely diagnosis?

a. Toxic epidermal necrolysis


b. Pemphigus vulgaris
c. Stevens-Johnson Syndrome
d. Bullous pemphigoid
e. Staphylococcal scalded skin syndrome

198. Which of the following best explains why oxygen delivery to the tissues is adequate at
rest in an acclimatized person at high altitude?

a. Arterial PO2 returns to normal


b. Hemoglobin concentration increases
c. The blood flow to the lungs increases, improving the V/Q ratio
d. Nitric oxide released from endothelial cells is increased
e. There is a compensatory decrease in the number of mitochondria

199. A 15-year-old boy brought by his parent to primary health care service with the chief
complaint of redness in both eyes. It happens for about a week. He intensely rubbing his
eye because of itching sensation. One of the ophthalmological finding was as the picture
below.

a. What is the clinical finding on the picture?


b. Conjunctival injection
c. Giant papil
d. Horner-Trantas dots
e. Keratoconus
f. Pseudogerontoxon

200. A 5-year-old boy came to the clinic with fever as a chief complain. The doctor noticed
that the lymph nodes are palpable from several locations.

Which of the following is most likely the normal histological characteristic for the
lymphatic organ above?

a. It is covered by a dense capsule


b. Afferent lymph vessels penetrate the capsule and empty into the subcapsular space
c. Medullary tissue is continuous from lobule to lobule throughout each lobe
d. Each lobulus is divided into the cortex, and the medulla
e. It consist of white pulp and red pulp

Вам также может понравиться